1
$\begingroup$

Given an arbitrary (i.e. not necessarily arc-length parameterised) planar parametric curve $C(t) = \Big(x(t), y(t)\Big)$, I'm looking to prove the formula for its (signed) curvature

$$\kappa = \frac{x'y'' - x''y'}{(x'^2 + y'^2)^\frac32}.$$

I would like to do so using perpendicular bisectors, as this seems the most natural approach to me.


My attempt starts as follows. Given three points on the curve $P = C(t-t_1)$, $Q = C(t)$ and $R = C(t+t_2)$, the intersection of the perpendicular bisectors of the edges connecting these points (see the illustration below) yields the centre of the circle going through $P$, $Q$ and $R$. In the limit, as both $t_1 \to 0$ and $t_2 \to 0$, the circle through the three points is known as the osculating circle with a radius $r = \frac{1}{\kappa}$.

enter image description here

The intersection of the perpendicular bisectors can be expressed as $S + \alpha u = T + \beta v$, with $S$ and $T$ the midpoints of the edges $PQ$ and $QR$, $\alpha$ and $\beta$ unknown scalar values, and $u$ and $v$ chosen to be the vectors $Q-P$ and $R-Q$ rotated $90$ degrees counter-clockwise. That is,

$$u = \Big( -(Q-P)_y, (Q-P)_x \Big) = \Big( y(t-t_1)-y(t), x(t)-x(t-t_1) \Big), \\ v = \Big( -(R-Q)_y, (R-Q)_x \Big) = \Big( y(t)-y(t+t_2), x(t+t_2) - x(t) \Big).$$

Re-expressing, we have $\alpha u - \beta v = T - S$, or

$$\left(\begin{array}{cc}y(t-t_1)-y(t) & y(t+t_2)-y(t) \\ x(t)-x(t-t_1) & x(t) - x(t+t_2)\end{array}\right) \left(\begin{array}{c}\alpha \\ \beta\end{array}\right) = \frac12 \left(\begin{array}{c}x(t+t_2) - x(t-t_1) \\ y(t+t_2) - y(t-t_1) \end{array}\right).$$

We can solve for $\alpha$ and $\beta$ by inverting this $2 \times 2$ matrix. Expressing the matrix entries symbolically, recall that

$$\left(\begin{array}{cc}u_x & -v_x \\ u_y & -v_y\end{array}\right)^{-1} = \frac{\left(\begin{array}{cc}-v_y & v_x \\ -u_y & u_x\end{array}\right)}{v_x u_y - u_x v_y}.$$

Taking the limit of $t_1 \to 0$ and $t_2 \to 0$, we should then get $r = \alpha \|u\| = \beta \|v\|$.


Unfortunately, this doesn't appear to result in the desired curvature formula (or well, its reciprocal). It probably means different lengths for the vectors $u$ and $v$ should be used — dividing them by $t_1$ and $t_2$, respectively, yields something that in the limit looks like the components of the tangent vector $C'(t) = \Big( x'(t), y'(t) \Big)$. However, that does not resolve the second derivatives appearing in the formula (which I suppose come from $T - S$). How to proceed?

$\endgroup$
2
  • $\begingroup$ Why make $t_1$ and $t_2 $ different? $\endgroup$ Commented Mar 11, 2022 at 21:31
  • $\begingroup$ @Buraian It felt like the right approach for curves that are not arc-length parameterised. Still, we could ultimately set $t_1 = t_2$. $\endgroup$ Commented Mar 12, 2022 at 9:40

2 Answers 2

1
$\begingroup$

You have $Q = C(t)$ and $R = C(t + h) $

The center of the circle passing through these two points satisfies

$(S - Q) \cdot (S - Q) = (S - R) \cdot (S - R) $

so that

$ S \cdot S - 2 S \cdot Q + Q \cdot Q = S \cdot S - 2 S \cdot R + R \cdot R $

from which

$ 2 S \cdot (R - Q ) = R \cdot R - Q \cdot Q \hspace{15pt}(1) $

Using Taylor series approximation of $R$, we get

$R = C(t + dt) = C(t) + h C'(t) + \frac{1}{2} h^2 C''(t) $

Plugging this into $(1)$ and retaining only differentials of first and second order, we get

$ 2 S \cdot ( h C' + \frac{1}{2} h^2 C'' ) = 2 h C \cdot C' + h^2 (C \cdot C'' + C' \cdot C' ) $

From this, it follows that

$ S \cdot C' = C \cdot C' \hspace{15pt}(2)$

and

$ S \cdot C'' = C \cdot C'' + C' \cdot C'\hspace{15pt}(3) $

Equation $(2)$ can be written as

$ (S - C) \cdot C' = 0 \hspace{15pt}(4) $

So that $(S - C)$ is perpendicular to $C'$. It follows that

$ S = C + \alpha R(90^\circ) C' \hspace{15pt} (5) $

Explicitly writing the components of $C= (x, y)$, $C' = (x', y')$, and $C'' = ( x'', y'' ) $, then $(5)$ becomes

$ S = (x - \alpha y' , y + \alpha x' ) $

Plugging this into $(3)$

$ x'' (x - \alpha y') + y'' (y + \alpha x') = x x'' + y y'' + x'^2 + y'^2$

Cancelling equal terms on both sides of the equation and solving for $\alpha$

$\alpha = \dfrac{ x'^2 + y'^2 }{ x' y'' - y' x'' } $

Now the radius of the circle is $\| S - Q \| = \alpha \| C' \| = \alpha \sqrt{x'^2 + y'^2 } $

Hence

$ r = \dfrac{ (x'^2 + y'^2)^{\frac{3}{2}} } { x' y'' - y' x''} $

and the curvature is

$ \kappa = \dfrac{1}{r} =\dfrac { x' y'' - y' x''}{ (x'^2 + y'^2)^{\frac{3}{2}} } $

$\endgroup$
4
  • $\begingroup$ Thanks, but this proof doesn't seem to be based on the intersection of the perpendicular bisectors. Moreover, you merely consider two points on the curve (i.e. $Q$ and $R$), which together lie on an infinite number of circles. Yet, you somehow end up with the (centre of the) circle I'm looking for... $\endgroup$ Commented Mar 13, 2022 at 16:08
  • 1
    $\begingroup$ True. It is not based on the intersection of perpendicular bisectors of two segments, because I am using only one segment $QR$. However, the length of the segment is varied by varying the step size $h$. And the center of the circle $S$ is kept constant as $h$ varies, and this is how it is derived. $\endgroup$ Commented Mar 13, 2022 at 17:53
  • 1
    $\begingroup$ What makes finding the center $S$ possible is the Taylor series expansion bases on $C(t)$ and its first and second derivatives. $\endgroup$ Commented Mar 13, 2022 at 17:59
  • $\begingroup$ I see — as a result of using the first- and second derivatives, the curve and circle have second-order contact (i.e. are osculating). $\endgroup$ Commented Mar 13, 2022 at 20:42
1
$\begingroup$

Using perpendicular bisectors we can define the osculating circle whose center $C$ is the center of curvature and whose radius is $\rho$.

The circle lying in the osculating plane at the curve at the point $P$, having radius $\rho$ is called the "osculating circle". We can show that: "the osculating circle is the limit figure of a circle passing through three points of the given curve, when they tend to coincide in only one. Let the center of this last circle be $C$, its radius $r$ and $P$, $P_{1}$, $P_{2}$ the three points that tend, in the passage to the limit, to coincide with $P$. Setting

$F(t)=(C-M)^{2}-r^{2}$,
which represents the equation of a circle of center $C$ and radius $r$ , passing through the generic point $M$.

Then the function $F(t)$ must be null when instead of $M$ we place $P$, $P_{1}$, $P_{2}$, since it is null when $M$ is a point of the circle of center $C$ and radius $r$.

By Rolle's theorem $F(t)$ is null at a point $P_{1}*$ between $P$ and $P_{1}$ and at a point $P_{2}*$ between $P_{1}$ and $P_{2}$;

and again for the same theorem $F''(t)$ is null at a point $P*$ between $P_{1}*$ and $P_{2}*$.

When $P_{1}$ and $P_{2}$ tend to $P$, $P_{1}*$, $P_{2}*$ and $P*$ also tend to $P$, then the center $C$ is defined by the equations:

$F(t)=0$, $F’(t)=0$, $F’’(t)$ ;

i.e.

$(C-P)^{2}=r^{2}$,

$(C-P)\dot{P}=0$,

$(C-P)\ddot{P}=\dot{P}^{2}$.

The first equation is the equation of the center circle $C$ and radius $r$, the second says that $C$ lies in the plane normal to the curve in $P$; therefore $C$ is on the main normal, because, having to be coplanar with $P$, $P_{1}$, $P_{2}$ at the limit, it must be on the osculating plane.

The third equation taking as a parameter the arc $s$, becomes:

$(C-P) \frac{1}{\rho} \vec{n}=1$,

i.e. remembering that $r$ is the module of $C-P$ e che $C-P$ ed $\vec{n}$ are parallel:

$r=\rho$.

At this point referring to Prove a relation for radius of curvature, using parametric coordinates

and in particular the formula:

$\frac{1}{\rho^{2}}= \frac{\ddot{x}^2+\ddot{y}^2-\ddot{s}^2}{\dot{s}^4}$

Placing in place of

$\ddot{s}=\frac{\dot{x}\ddot{x}+\dot{y}\ddot{y}}{\dot{s}}$,

and in place of

$\dot{s}^2=\dot{x}^2+\dot{y}^2$,

we get:

$\frac{1}{\rho}=\frac{\dot{x}\ddot{y}-\dot{y}\ddot{x}}{\Big(\dot{x}^2+\dot{y}^2\Big)^{\frac{3}{2}}}$.

$\endgroup$
1
  • $\begingroup$ This proof reminds me of Alfred Gray's one in Modern Differential Geometry of Curves and Surfaces (Theorem 4.12). However, it does not seem to be based on (the intersection of) the perpendicular bisectors (even though you mention them at the start of your answer). $\endgroup$ Commented Mar 16, 2022 at 10:27

You must log in to answer this question.

Start asking to get answers

Find the answer to your question by asking.

Ask question

Explore related questions

See similar questions with these tags.